Final&Exam&Solutions& - University of Florida · PHY2049Spring2012$ $ Final$Examsolutions$...

13
PHY 2049 Spring 2012 Final Exam solutions Final Exam Solutions Note that there are several variations of each problem, indicated by choices in parentheses. 1. A graph of the x component of the electric field as a function of x in a region of space is shown in figure. The scale of the vertical axis is set by Exs = 40.0 (or 30 or 70) N/C. The y and z components of the electric field are zero in this region. If the electric potential at the origin is 30 V, what is the electric potential (in V) at x = 4.0 m? (1) -20 (2) 0 (3) -80 (4) 140 (5) 60 The change in the electric potential from the origin to x=4 is given by: !V = " E dx x 1 x 2 # = " 1 2 E xs 4 = "80V , "60V , "140V So the potential at x = 4 is 60 + ΔV = 20 V, 0 V, 80 V for the 3 possibilities. 2. In the shown figure, a potential difference of V = 12 V is applied across the arrangement of capacitors with capacitances of C1 = C2 = 2μF, and C3 = 3 μF. What is the charge q2 on capacitor C2 (or the charge q3 on C3)? (1) 12 μC (2) 36 μC (3) 48 μC (4) 24 μC (5) 6 μC

Transcript of Final&Exam&Solutions& - University of Florida · PHY2049Spring2012$ $ Final$Examsolutions$...

PHY  2049  Spring  2012     Final  Exam  solutions  

Final  Exam  Solutions    Note  that  there  are  several  variations  of  each  problem,  indicated  by  choices  in  parentheses.    1.  A  graph  of  the  x  component  of  the  electric  field  as  a  function    of  x  in  a  region  of  space  is  shown  in  figure.  The  scale  of  the  vertical  axis  is  set  by  Exs  =  40.0  (or  30  or  70)  N/C.  The  y  and  z  components  of  the    electric  field  are  zero  in  this  region.  If  the  electric  potential  at  the  origin  is  30  V,  what  is  the  electric  potential  (in  V)  at  x  =  4.0  m  ?  

(1) -20 (2) 0 (3) -80 (4) 140 (5) 60

The  change  in  the  electric  potential  from  the  origin  to  x=4  is  given  by:    

!V = " Edxx1

x2# = " 12Exs4 = "80V ,"60V ,"140V  

    So  the  potential  at  x  =  4  is  60  +  ΔV  =  -­‐20  V,  0  V,  -­‐80  V    for  the  3  possibilities.      2.  In  the  shown  figure,  a  potential  difference  of  V  =  12  V  is  applied  across  the  arrangement  of  capacitors  with  capacitances  of  C1  =  C2  =  2μF,  and    C3  =  3  μF.  What  is  the  charge  q2  on  capacitor  C2  (or  the  charge  q3  on  C3)?      

     (1)  12  μC    (2)    36  μC    (3)    48  μC    (4)    24  μC    (5)    6  μC    

PHY  2049  Spring  2012     Final  Exam  solutions  

  The  equivalent  capacitance  is    

Ceq =1C1

+ 1C2

!"#

$%&

'1

+C3 = 4µF  

  The  total  charge  stored  on  the  capacitors  is:       qtot = CeqV = 4µF( ) 12 V( ) = 48µC       That  on  C3  is  similarly:       qtot = CeqV = 3µF( ) 12 V( ) = 36µC       So  the  difference  is  what  is  on  q2,  which  is  also  what  is  on  q1  by  the  way:      

q2 = qtot ! q3 = 12µC    Problem  3    Two  straight  parallel  power  lines  are  separated  by  a  distance  of  2m.  Each  carries  a  current  of  5000  A,  in  the  same  (or  opposite)  direction.  What  is  the  magnitude  of  the  force  per  unit  length  on  the  wires,  and  is  the  force  attractive  or  repulsive?    (1)  2.5  N/m,  attractive  (2)  2.5  N/m,  repulsive  (3)  5  E-­‐4  N/m,  repulsive  (4)  5  E-­‐4  N/m,  attractive  (5)  1.3  E7  N/m,  repulsive    The  force  between  two  wires  carrying  electric  currents  is:  

FBA = µ0iAiBL2!d  

 The  magnitude  of  the  force  per  unit  length  is  therefore:    FL=4!10"7( ) 5000( )2

2!( ) 2( ) = 2.5N    

 To  get  the  sign  of  the  force:      

• parallel  wires  with  currents  in  the  same  direction  feel  an  attractive  force.    • Wires  with  currents  in  the  opposite  direction  feel  a  repulsive  force.    

     

PHY  2049  Spring  2012     Final  Exam  solutions  

Problem  4    What  is  the  flux  leaving  the  surface  of  the  shown  cube  if  the  electric  field  is  given  by  

!E = 2xi ! 3yj    (or  

!E = 3xi ! 2yj    or  

!E = 4xi ! 6yj )  and  the  cube  has  a  side  length  of  

2?  

 (a)    -­‐8                                                (b)    8                                                    (c)    -­‐16                                              (d)    16                                              (e)  0    

Because  of  the  directions  of  the  electric  field,  the  flux  could  only  be  non-­‐zero  for  the  top-­‐bottom  and  left-­‐right  faces.  However,  because  the  field  also  depends  on  x  and  y,  the  only  non-­‐zero  contributions  are  for  the  top  and  right  faces:  

! = !R +!T = ER "AR +ET "AT

= (2)(2)(2)2 # (3)(2)(2)2 = #8 or= (3)(2)(2)2 # (2)(2)(2)2 = 8 or= (4)(2)(2)2 # (6)(2)(2)2 = #16  

    Recall  that  flux  is  positive  when  the  field  points  out  from  a  closed  surface.      Problem  5    In  the  circuit  shown,  the  ideal  batteries  have  EMFs  of    ε1  =  9  V  and  ε2  =  4.5  V  and  the  resistances  are  R1=30  Ω  and  R2=15  Ω.  If  the  potential  at  Q  (or  instead  P)  is  defined  to  be  3  V,  what  is  the  potential  at  P  (or  instead  Q)?    

 (1)  10.5  V    (2)    -­‐4.5  (3)    7.5  V    (4)    4.5  V    (5)    9  V            

PHY  2049  Spring  2012     Final  Exam  solutions  

First  solve  for  the  current  in  the  circuit:    

!1 " iR2 " !2 " iR1 = 0

i = !1 " !2R1 + R2

= 4.545

= 0.1    

    Now  let’s  add  the  potential  difference  to  go  from  point  Q  to  P  (or  P  to  Q):    

 

VP =VQ + !1 " iR2VP = 3+ 9 " (0.1)15 = 10.5orVQ =VP " !2 " iR1VQ = 3" 4.5 " (0.1)30 = "4.5

   

   Problem  6    A  50-­‐turn  (or  25  turn)  circular  coil  of  wire  has  a  diameter  of  2m.  It  is  placed  with  its  axis  aligned  with  a  magnetic  field  of  10-­‐4  T.  Then  the  coil  is  flipped  180°  in  0.2s.  An  average  EMF  of  what  magnitude  is  generated  in  the  coil?    (1)  160mV    (2)  80mV      (3)  3.1mV      (4)  1.6mV        (5)  31mV    Induced  EMF  is    

E = !"!t

= B#r2N $ ($B#r2N )!t

= 2B#r2N!t

=2 %10$4 %3.14 % 2

2&'(

)*+

2

% 50 or 25( )0.2

= 0.16V or 0.08V  

   

PHY  2049  Spring  2012     Final  Exam  solutions  

Problem  7    A  glass  rod  forms  a  semi-­‐circle  of  radius  r    =  7  cm  with  a  charge  of  +q    (or  –q)  distributed  uniformly  along  the  upper  quadrant  and  –q  (or  +q)    distributed  along  the  lower  quadrant,  where  q    =  5  pC.  What  is  the  magnitude  and  the  direction  (as  the  polar  angle  relative  to  the  direction  of  the  x  -­‐axis)  of  the  electric  field  at  the  center  P    of  the  semi-­‐circle.  

 (1)  12  N/C,    θ=  270°  (2)  12  N/C,  θ  =  90°  (3)  0  N/C  (4)  18  N/C,  θ  =  180°  (5)  18  N/C,  θ  =  0°    The  diagram  shows  the  direction  of  the  electric  field  contribution  from  each  quadrant.  By  symmetry,  the  net  field  points  down  in  the  ! j  direction  (or   i    direction),  which  is   ! = 270!  (or   ! = 90! )    The  magnitude  (not  really  needed  to  choose  the  correct  answer!)  is  given  by:    

! = q"2R

dq = !ds = !Rd#

Ey+q = k dq

R2sin# =

" /2

"

$ k !Rd#R2

sin#" /2

"

$= %k !

Rcos# |" /2

" = k !R

& Eytot = 2k !

R= 4k

"qR2

= 12  

         

PHY  2049  Spring  2012     Final  Exam  solutions  

Problem  8    An  electron  moves  in  the   i    direction,  through  a  uniform  magnetic  field  in  the  ! j    (or  !k  )  direction.    The  magnetic  force  on  the  electron  is  in  the  direction:    (1)     k      (2)  ! j      (3)     j    (4)    !k      (5)    -­‐i       The  magnetic  force  is  FB = qv !B     So  working  with  directions  only:    

 F = !e( ) i( )" ! j( ) = k or

F = !e( ) i( )" !k( ) = ! j  

   Problem  9    Four  charges  are  evenly  spaced  along  the  x    axis  with  a  separation  distance  d    =  3  cm.  The  values  of  the  charges  are:  q1    =  +4    μC,  q2    =  -­‐  2  μC,  q3    =  +2  μC,  and  q4    =  +6  μC.  What  is  the  net  electrostatic  force  along  the  x  -­‐axis  acting  on  charge  q1    due  to  the  other  charges?  

   (1)  33  i    N    (2)      -­‐240  i  N      (3)    130  i    N      (4)  -­‐27  i    N    (5)  80  i    N    Carefully  work  out  the  magnitude  and  direction  of  the  force  between  charge  q1  and  the  others:    F1 = F12 + F13 + F14

= +kq1q2

d 2 i ! kq1q3

2d( )2 i ! kq1q4

3d( )2 i

=9 "109( ) 10!12( )

0.03( )2 8 ! 84! 24

9#$%

&'( i = 33 i N

 

   

PHY  2049  Spring  2012     Final  Exam  solutions  

Problem  10    In  the  figure  shown,  the  ideal  batteries  have  EMFs  ε1  =  9.0V  and  ε2  =  3.0V,  and  the  resistances  are  each  4Ω    What  is  the  magnitude  of  the  current  in  resistor  3?  (1)  1.0A      (2)  1.25      (3)  0.25A      (4)  0.5A      (5)  0A    

   Kirchoff’s  junction  rule  tells  us  i3 = i1 + i2      Kirchoff’s  loop  rule  for  the  left  loop  gives:  !1 " i1R1 " i3R3 = 0      Kirchoff’s  loop  rule  for  the  right  loop  gives:  !2 " i2R1 " i3R3 = 0      Let’s  add  these  two  equations:    !1 + !2 " i1 + i2( )R1 " 2i3R3 = 0!1 + !2 " i3R1 " 2i3R3 = 0!1 + !2 " 3i3R3 = 0

i3 =!1 + !23R3

= 9 + 33#4

= 1.0

 

       

PHY  2049  Spring  2012     Final  Exam  solutions  

Problem  11    A small spaceship with a mass of only 1.0×103 kg (including an astronaut) is at rest in outer space with negligible gravitational force acting on it. If the astronaut turns a 10 kW laser beam, what speed will the ship attain in 1 day because of the momentum carried away by the beam? Solution: The total amount of energy carried away by the laser beam is U = I ! t  

This  corresponds  to  momentum   p = 1c!U = 1

c! I ! t  

From  momentum  conservation,  the  spaceship  will  acquire  the  same  amount  of  momentum  (pointing  away  from  the  direction  of  the  laser  beam).  The  velocity  corresponding  to  this  momentum  is  then:    

v = pm

= 1mc

! I ! t = 104 J !(24 !60 !60s)103kg !3"108m / s

= 0.00288 m / s

     Problem  12    A  parallel  plate  capacitor  has  circular  plates  of  radius  R  and  plate  separation  d.  The  potential   difference   between   the   plates   is   given   by   the   time   dependent   function.  What   is   the  maximum  value  of   the  magnetic   field   induced  between   the  plates  at  a  radial  distance  2R  from  the  axis  of  symmetry?      Solution:

The electric field between the capacitor plates at any given time is E = Vd= Edsin(2! f t)  

The  flux  of  the  filed  through  the  circle  of  radius  2R  (note:  outside  of  radius  R,  there  

is  no  field):  !E = E(!R2 ) = E (!R

2 )d

sin(2! f t)  

Then,  we  can  use  !B !d!s"" = 1

c2d#E

dt:        B !2! (2R) = 1

c2E (!R2 )

d(2! f )cos(2! f t) ,  

From  where  the  amplitude  of  the  magnetic  field  is  B = 1c2E !Rf2d

 

       

PHY  2049  Spring  2012     Final  Exam  solutions  

Problem  13    An oscillating current in an inductor-capacitor LC-circuit emits electromagnetic waves with a wavelength λ. What is this wavelength, if L=0.25 µH and C=25 pF? Solution:

The frequency of oscillation in LC-circuit is f = 12!

1LC

 

The  wavelength  is  then  ! = c / f = 12"

1LC

= 2" LC !c = 4.71 m  

 Problem  14    

Initially unpolarized light is sent along the z-axis into a system of three polarizing sheets placed perpendicular to the z-axis and whose polarizing angles with respect to y-axis are 30º (first sheet on the way of light), 90º (second sheet on the way of light), and 45º (the last sheet). What percentage of the initial light intensity is transmitted by the system?

Solution: Fraction of unpolarized light transmitted by the first polarizing sheet is  ½.        The  transmitted  light  is  now  polarized.    The  second  polarizing  sheet  is  rotated  with  respect  to  this  light  by  90º -­‐  30º =  60º. The fraction of light transmitted is the second polarizing sheet is cos2(60º) = ¼ The light is again polarized and the plain of polarization is now at 90º. The third polarizing sheet is rotated by 45º with respect to the plain of polarization of the incoming light. The fraction of light transmitted is the third polarizing sheet is cos2(45º) = ½. The final fraction of the light intensity that makes through all three polarizing sheets is

then 12! 14! 12= 116

" 6%  

   

PHY  2049  Spring  2012     Final  Exam  solutions  

Problem  15    A  ray  of  light  enters  a  glass  cube  from  air  at  point  A  at  an  incident  angle  of  θ=30o  and  then  undergoes  total   internal   reflection   at   point   B.     What   the  minimum  value  of  index  of  refraction  n  of  glass  can  be  inferred  from  this  information?          (a)  1.12                            (b)  1.22                          (c)  1.32                            (d)  1.39                          (e)  1.41            Solution: By Snell’s law, the refracted light at point A will have an angle θR with respect the normal to the cube side surface such  that      sinθ  =  n  ⋅  sinθ R,  or  sinθ R=  sinθ  /  n    The refracted ray will make an incident angle with respect the top surface (90º-θR). For the full reflection: sin(90º-θR) ≥ 1/n,  from  where:    cosθR ≥ 1/n By squaring and adding the two equations one obtains (remember that sin2θ R+cos2θ R=1):

1! sin2!n2

+ 1n2

n ! 1+ sin2! = 1+ sin2 30° = 1.25 =1.12

       

PHY  2049  Spring  2012     Final  Exam  solutions  

Problem  16    Two identical converging lenses with a focal distance of f=8 cm are separated by a distance 16 cm. An object of 1-cm size is placed 4 cm in front of the two-lens package. What is the absolute size of the image formed by the two lenses? Solution: The first lens makes an image at the distance i1 = -8 cm, which we find from 1p1

+ 1i1= 1f.  

This  image  is  virtual,  i.e.  it  is  in  front  of  the  first  lens.    The  image  is  magnified  by  m1  =  |i1/p1|  =  2  times.        The  image  of  the  first  lens  is  the  object  for  the  second  lens  at  a  distance  p2=16  cm  away  from  it.  The  image  produced  by  second  lens  then  appears  at  distance  i2 = 8 cm, which we find from 1p2

+ 1i2= 1f.

The  magnification  produced  by  the  second  lens  is  m2  =  |i2/p2|  =  0.5.    The  final  magnification  is    m=  m1  ⋅  m2  =  2  ⋅  0.5  =1.    Therefor,  the  final  image  is  of  the  same  size  as  the  original  object,  i.e.  1  cm.    

PHY  2049  Spring  2012     Final  Exam  solutions  

Problem  17    An object is placed 3.6 cm in front of a concave/convex mirror. Its image is virtual and four times tall as the object. Find the focal length of the mirror.  (a)  4.8  cm   (b)  -­‐4.8  cm     (c)  2.4  cm   (d)  -­‐2.4  cm   (e)  1.6  cm    Solution: 1p+ 1i= 1f

The image is virtual (i is then negative) and four times larger than the object (|i/p|=4).

Therefore, 1p! 14p

= 1f.    From  here,   f = 4

3p = 4.8 cm  

     Problem  18    Two  coherent  radio-­‐frequency  point  sources  separated  by  d=4  m  radiate  in  phase  with  a  wavelength  λ=1  m.  A  detector  moves  in  a  circular  path  of  some  radius  R  around  the  mid-­‐point  between  the  two  sources  (see  figure)  and  measures  the  radio-­‐wave  intensity  of  the  signal.  Find  how  many  maxima  it  detects.        Solution:

The  phase  difference  between  two  sources  at  the  top  point  is  !! = 2" !L#

= 2" "4  

The  phase  difference  between  two  sources  at  the  top  point  is  !! = 2" !L#

= 2" "(#4)  

The  phase  difference  between  two  sources  at  leftmost  and  rightmost  top  points  !! = 0 = 2" "(0)    The  condition  for  the  maxima  points  is  !! = 2" "m ,  where  m  is  any  integer  number.  As  a  detector  moves  along  the  circle,  the  phase  difference  changes  from  !! = 2" "(#4)  to  !! = 2" "4 .  Hence,  if  one  starts  at  the  bottom,  m  changes  from  -­‐4  (bottom)  to  4  (top)  and  back  to  -­‐4  (bottom).  The  number  of  integers  in  this  sequences  is  16:  -­‐4,  -­‐3,  -­‐2,  -­‐1,  0,  1,  2,  3,  4,  3,  2,  1,  0,  -­‐1,  -­‐2,  -­‐3,  -­‐4.      Hence,  the  number  of  maxima  occurring  along  the  circle  is  16.    

PHY  2049  Spring  2012     Final  Exam  solutions  

 Problem  19    We  wish  to  coat  a  glass  lens  (n=1.50)  with  a  transparent  material  (n=1.25)  so  that  the  reflection  of  light  at  wavelength  600  nm  (in  air)  is  eliminated  by  interference.  What  minimum  thickness  can  the  coating  have  to  do  this?    Solution: The  ray  that  bounces  off  the  coating  acquires  !1 = " (reflection  from  a  higher-­‐index-­‐of-­‐refraction  material).    The  ray  that  goes  into  the  coating  material,  bounces  off  glass  surface,  and  then  goes  

back  through  the  coating  acquires  !2 = 2"h#n

+" + 2" h#n,    

where  !n =!n  is  the  wavelength  of  light  inside  the  material  of  coating  (n=1.25).  

To  eliminate  reflection,  the  interference  between  the  two  ray  must  be  destructive,  i.e.  !! =!2 "!1 = " + 2"m    

Hence,   2! h"n

+! + 2! h"n

!"#

$%&'! = ! + 2!m ,  from  where  

h = 1+ 2m4

!n =1+ 2m

4!n

hmin =!4n

=120 nm  

   Problem  20    What  is  the  smallest  object  on  the  Moon  that  can  be  resolved  with  the  Hubble  Space  Telescope  whose  diameter  is  2.4  m?  The  typical  light  wavelength  is  500  nm.  The  distance  between  the  Earth  and  the  Moon  is  400,000  km.  Neglect  the  fact  that  the  telescope  is  at  600  km  above  the  Earth.    Solution:    Telescope  angular  resolution  due  to  the  diffraction  of  light  on  the  aperture  of  the  mirror  is  1.22⋅λ/D,  where  D  is  the  mirror  diameter.    An  object  of  size  h  at  distance  d  has  an  angular  size  h/d.    From  making  the  two  equal,  h/d  =  1.22  ⋅  λ/D,  one  finds  h  =  1.22  ⋅  λ/D  ⋅  d  =  102  m